Money that is collecting interest in a bank account doubles in value every 10 years. When the account was opened, it had 16, or 2* dollars in it. After 50 years, the account balance has doubled 5 times, increasing by a factor of 2 It now has a balance of 2*.25. Expressed as a power, how much money, in dollars, is in the account after 50 years? 1

Answers

Answer 1

The amount in dollars that is in the account after 50 years expressed in power is; 2⁹

How to apply the rule of exponents?

We are told that the money doubles in value after every 10 years.

When the account was opened it had 2⁴ dollars in it and after 50 years it has doubled 5 times. The doubling increased by a factor of 2⁵ and is now has a balance of 2⁴ × 2⁵.

Using the rule of exponents, we have;

2⁴ × 2⁵ = 2⁹

Read more about Rule of Exponents at; https://brainly.com/question/11761858

#SPJ1


Related Questions

What is the volume of the solid?

Answers

Answer: C. 3052,08 cm³.

Step-by-step explanation:

D=18 cm       V=?

[tex]R=\frac{D}{2} =\frac{18\ cm}{2} =9\ cm.\\\boxed {V=\frac{4}{3}*\pi *R^3 }\\V=\frac{4}{3}*\pi *9^3 \\V=4*\pi *3*9^2\\V=12*81*3,14 \\V=3052,08\ (cm^3).[/tex]

How many solutions exist for the given equation? 0.75(x 40) = 0.35(x 20) 0.35(x 20)

Answers

The equation 0.75(x+40)=0.35(x+20)+0.35(x-20) is having only one solution.

Given equation 0.75(x+40)=0.35(x+20)+0.35(x-20)

We have to find the number of solutions.

To find the number of solutions we have to solve the equation for the value of variable x. Equation is often solved to find the value of variables.

Equation : 0.75(x+40)=0.35(x+20)+0.35(x-20)

Opening the brackets and multiply the numbers outside by the numbers in the bracket.

0.75x+30=0.35x+7+0.35x-7

Take all the numbers having same variables at one end of equal to.

0.75x-0.35x-0.35x=7-7-30

0.75x-0.7x=-30

0.05x=-30

x=-30/0.05

x=-600

Which is the only value of x.

Hence the number of solutions of the equation 0.75(x+40)=0.35(x+20)+0.35(x-20) is  one.

Learn more about equation at https://brainly.com/question/2972832

#SPJ4

Can someone help? ASAP Find each missing length, give all answers in simplest radical form

Answers

I don’t know the answer I need point because this app makes me answer these when I don’t know them just for me to get them wrong and not be able to do them correct and fail just y=mx+b it is 6 and 7

A machine that is programmed to package 1.50 pounds of cereal is being tested for its accuracy in a sample of 56 cereal boxes, the sample mean filling weight is calculated as 1.52 pounds. The population standard deviation is known to be 0.06 pounds. Find the 95% confidence interval for the mean.

Answers

The 95% confidence interval for the mean is (1.535, 1.504).

It is given that a machine produces 1.50 pounds of cereal. The sample is 56 cereal boxes. The sample mean filling weight is calculated as 1.52 pounds. The population standard deviation is known to be 0.06 pounds.

Mean = 1.50 pounds

x = 1.52 pounds

Standard deviation = SD = 0.06 pounds

n = 56

We need to find the 95% confidence interval for the mean.

So, z = 1.96

So, the confidence interval would be

[tex]x[/tex]±[tex]z\frac{mean}{\sqrt{n} }[/tex]

[tex]1.52[/tex]±[tex]1.96*\frac{0.06}{\sqrt{56} }[/tex]

= [tex]1.52[/tex]±[tex]1.96*0.00801[/tex]

= [tex](1.52 + 0.0156996),(1.52 - 0.0156996)[/tex]

= [tex](1.535, 1.504)[/tex]

To learn more about binomial distribution visit: brainly.com/question/25096507

#SPJ4

Volume=
What is the volume?
Please help thank you

Answers

904.78 if you have to round it’s 904.8

Answer:

288π

Step-by-step explanation:

The volume of a sphere can be evaluated by using the following formula:

[tex]A=\frac{4}{3}\pi r^{3}[/tex]

We are given that the radius is 6, so we can plug into the formula to solve:

[tex]A=\frac{4}{3}\pi*6^{3} =\frac{4}{3}*216*\pi =288\pi[/tex] (216/3 is 72, and 72*4 is 288)

Therefore, the volume is exactly 288π. The approximate answer, if you let π be 3.14 and evaluate it out, is around 904.78

The names of the months are each written on a slip of paper. if a slip of paper is randomly drawn, which scenario depicts mutually exclusive events? begins with a j and contains a u ends with an r and has five or fewer letters has thirty-one days and ends with a y contains an e and has thirty or fewer days

Answers

Answer:

Ends with r and has five or fewer letters

Step-by-step explanation:

Answer:

The answer is : B

Step-by-step explanation:

trust me I did it on edge 2022

Find the length of JH help pls

Answers

Answer:

c) 19

Step-by-step explanation:

To find the length of JH, subtract the length of HM from JM.

Length of JH = JM - HM

                      = 40 - 21

                      = 19

Answer:

JH = 19

Step-by-step explanation:

from the diagram

JH + HM = JM , that is

JH + 21 = 40 ( subtract 21 from both sides )

JH = 19

1. Try It #1
If f(x) is a linear function, and (2,3) and (0,4) are points on the line, find the slope. Is this function increasing or decreasing?

Answers

Answer:

The slope of the line representing the linear function f(x) is determined as [tex]$-\frac{1}{2}$[/tex]. The function f(x) is decreasing because the slope is less than zero, i.e., m<0.

Step-by-step explanation:

A linear function f(x) is given with two points, (2,3) and (0,4) lying on the line representing f(x).

It is asked to determine the slope of the line and state if the function is increasing or decreasing. of the value of the slope obtained.

Step 1 of 1

Determine the slope of the line.

The points as given in the question are (2,3) and (0,4). Now, the formula for the slope is given as

[tex]$m=\frac{y_{2}-y_{1}}{x_{2}-x_{1}}$[/tex]

So, substitute [tex]$4 \& 3$[/tex] for [tex]$y_{2}$[/tex] and [tex]$y_{1}$[/tex] respectively, and [tex]$0 \& 2$[/tex] for [tex]$x_{2}$[/tex] and [tex]$x_{1}$[/tex] respectively in the above formula. Then simplify to get the slope as follows,[tex]$m=\frac{4-3}{0-2}$\\ $m=\frac{1}{-2}$\\ $m=-\frac{1}{2}$[/tex]

The slope of the line is obtained as [tex]$-\frac{1}{2}$[/tex]. Now, as [tex]$-\frac{1}{2} < 0$[/tex], so the function f(x) is decreasing.

There are 135 people in a sport centre.
77 people use the gym.
62 people use the swimming pool.
65 people use the track.
27 people use the gym and the pool.
23 people use the pool and the track.
31 people use the gym and the track.
4 people use all three facilities.
Given that a randomly selected person uses the gym and the track, what is the probability they do not use the swimming pool?

Answers

The probability that they do not use the swimming pool will be 0.2.

How to calculate the probability?

From the information given, there are 135 people in a sport centre and 31 people use the gym and the track while 4 people use all three facilities.

Based on the information given, the people who use the gym and the track only will be:

= 31 - 4

= 27

The probability that they do not use the swimming pool will be:

= 27/135

= 0.2

Learn more about probability on:

brainly.com/question/24756209

#SPJ1

A principal amount of $3000 is invested for 3 years at 4% compounded semi-annually. Use the compound interest formula to determine how much interest was earned on the investment.

Answers

The $378.49 was earned on the investment if the principal amount of $3000 is invested for 3 years at 4% compounded semi-annually.

What is compound interest?

It is defined as the interest on the principal value or deposit and the interest which is gained on the principal value in the previous year.

We can calculate the compound interest using the below formula:

[tex]\rm A = P(1+\dfrac{r}{n})^{nt}[/tex]

Where A = Final amount

          P = Principal amount

          r  = annual rate of interest

          n = how many times interest is compounded per year

          t = How long the money is deposited or borrowed (in years)

Here,

P = $3000

r = 4% = 0.04

n = 2

t = 3 years

After plugging all the values in the formula:

[tex]\rm A = 3000(1+\dfrac{0.04}{2})^{2\times 3}[/tex]

After solving:

A = $3,378.49

I = A - P = 3,378.49 - 3000 = $378.49

Thus, the $378.49 was earned on the investment if the principal amount of $3000 is invested for 3 years at 4% compounded semi-annually.

Learn more about the compound interest here:

brainly.com/question/26457073

#SPJ1

help me help me ………….

Answers

Answer:

not similar

Step-by-step explanation:

not similar--it doesn't work out for the angles to be similar

if we find the angle x of the right-hand triangle, we can tell it would be 41 degrees

meaning that these triangles are not similar

HELPPPP ASAP !!

Write a compound inequality that represents the given graph

Answers

Answer:

x <= -2 or x >= 3

Step-by-step explanation:

Shaded circles represent <= or >=, and the first arrow points left from -2, and the second arrow points right from 3. Therefore x <= -2 or x >= 3 would represent the graph.

a customer buys 3 cappuccinos that costs $2.54 each and a bottle of water that costs $0.76 she gives the shop assistant $20 how much change should she receive​

Answers

Answer:

$11.62

Step-by-step explanation:

2.54×3=$7.62

0.76+7.62=8.38

20-8.38=$11.62

Answer:

Step-by-step explanation:

Discussion

3 cappuccinos at 2.54 each     2.54 * 3 = 7.62

1 bottle of water at 0.76              0.76*1  =   .76     Add

Total                                                             8.38

Change = 20 - 8.38 = 11.62

Answer

She should receive 11.62 in change.

Jamaal washes 25 dishes per hour and works for 3 hours. Daniel can wash 22 dishes every two hours. Who can wash more dishes in 3 hours? And by how many?​

Answers

Answer:

1. Jamaal

2. 42 more dishes

Step-by-step explanation:

1. Jamaal: 25 dishes per hour and works for 3 hours

25(3) = 75 dishes

Daniel: Daniel can wash 22 dishes every two hours.

22 = 2x

11 = x

11(3) = 33 dishes

2. 75 - 33 = 42

Jamaal can wash 42 more dishes than Daniel.

Jamaal washes 25 dishes per hour and works for 3 hours. Daniel can wash 22 dishes every two hours. Jamaal washed 42 more dishes than Daniel in 3 hours.

Let's calculate the number of dishes washed by each person in 3 hours and then compare the results.

Jamaal:

Jamaal washes 25 dishes per hour and works for 3 hours.

Number of dishes washed by Jamaal in 3 hours = 25 dishes/hour * 3 hours = 75 dishes

Daniel:

Daniel washes 22 dishes every two hours.

To find the number of dishes washed by Daniel in 3 hours, we need to find how many times he washes dishes in 3 hours.

Number of times Daniel washes dishes in 3 hours = 3 hours / 2 hours (time for 1 cycle) = 1.5 cycles

Now, multiply the number of cycles by the number of dishes washed per cycle:

Number of dishes washed by Daniel in 3 hours = 1.5 cycles * 22 dishes/cycle = 33 dishes

Comparison:

Jamaal washed 75 dishes in 3 hours.

Daniel washed 33 dishes in 3 hours.

Now, let's find the difference in the number of dishes washed by each person:

Difference = Jamaal's dishes - Daniel's dishes

Difference = 75 dishes - 33 dishes = 42 dishes

Therefore, Jamaal washed 42 more dishes than Daniel in 3 hours.

To know more about  hours here

https://brainly.com/question/21289653

#SPJ2

How many ways can the team assign field positions to 9 of the 22 players, putting each of the 9 selected players in a position he can play, and ensuring that all 9 field positions are filled

Answers

The number of ways is 497420

The ways can the team assign field positions to 9 of the 22 players, putting each of the 9 selected players in a position he can play, and ensuring that all 9 field positions are filled is [tex]22_{C_9}[/tex]

A combination is a mathematical technique that determines the number of possible arrangements in a collection of items where the order of the selection does not matter. In combinations, you can select the items in any order.

22C9 = 22! /(9!*13!) = 497420

Thus, In 497420 ways the team assign field positions to 9 of the 22 players, putting each of the 9 selected players in a position he can play, and ensuring that all 9 field positions are filled.

Learn more about Combinations:

https://brainly.com/question/4313883

#SPJ4

i have no clue what to do

Answers

Answer:

4x + 3y = 12

Step-by-step explanation:

Answer:

4x + 3y = 12

Step-by-step explanation:

At x-intercept, y = 0.

∴ When we substitute y = 0 into the equation, x should equal 3.

• 1st equation:     3x + 4(0) = 12  ⇒  x = 4       ❌

• 2nd equation:  4x + 3(0) = 7    ⇒  x = 7/4    ❌

• 3rd equation:   3x + 4(0) = 7    ⇒  x = 7/3    ❌    

• 4th equation:   4x + 3(0) = 12   ⇒  x = 3       ✅

This means the 4th option is the correct one.

To check, we can replace x = 0 into the 4th equation to see if the y-intercept value comes out to be 4:

4(0) + 3y = 12    ⇒    y = 4     ✅

sk...
What is as for the geometric sequence 6, 561; -2, 187; 729; -243;....
A. 1
B. -3
C. -9
D. -1

Answers

Option B is the answer. The term for the geometric sequence that we have here is -3.

How to solve for the geometric sequence

We have u1, u2, u3 = a, ar, ar², ar³

From here we have to find the common ratio that is used to divide the sets of numbers

6,561 divided by −2,187 = -3

Hence when we go further each of the value when divided by -3 gives the next value.

Hence the value for the geometric sequence is -3.

Read more on geometric sequence here:

https://brainly.com/question/1509142

#SPJ1

What substitution should be used to rewrite 6(x + 5)2 + 5(x + 5) -4 = 0 as a quadratic equation?
Ou= (x + 5)
Ou= (x-5)
u = (x + 5)²
O u =(x-5)²

Answers

The substitution that should be used to rewrite the equation is u = x+5, the correct option is A.

What is a Quadratic Equation?

The polynomials of the 2nd degree are called Quadratic Equations.

It has a general form as ax² +bx+c=0

To rewrite the expression as a quadratic equation  

6(x+5)² + 5(x+5)-4 = 0

=6(x+5)(x+5)+5(x+5)-4

=6(x² + 5x + 5x + 25)+5(x+5)-4

=6(x²+10x+25) + 5(x+5) - 4 = 0

=6x² + 60x + 5x + 150 + 25 - 4 = 0

=6x² + 65x + 171 = 0

This is the quadratic equation of the form ax² +bx+c=0

but if directly it has to be written in this ax² +bx+c=0 form , it can be written by substituting u = (x + 5)

6(x+5)² + 5(x+5)-4 = 0

6u²+5u-4 = 0

Therefore, the substitution that should be used to rewrite the equation is u = x+5, the correct option is A.

To know more about Quadratic equations

https://brainly.com/question/2263981

#SPJ1

the measures of the sides of a triangle are 2x+3, 8x-4 and x^2. Write an expression that describes the perimeter of the triangle

Answers

Answer:

[tex]\textsf{Perimeter}=x^2+10x-1[/tex]

Step-by-step explanation:

The perimeter of a two-dimensional shape is the distance all the way around the outside.

Given sides of a triangle:

[tex]2x + 3[/tex][tex]8x - 4[/tex][tex]x^2[/tex]

Therefore, to find the perimeter of the triangle, sum the given sides:

[tex]\begin{aligned}\sf Perimeter & = (2x+3)+(8x-4)+x^2\\& = 2x+3+8x-4+x^2\\& = x^2 + 2x+8x+3-4\\& = x^2+10x-1\end{aligned}[/tex]

Perimeter=

Sum of sidesx²+2x+3+8x-4x²+2x+8x+3-4x²+10x-1

A population of squirrels is growing in a Louisiana forest with a monthly growth constant of 55 percent. If the initial count is 100100 squirrels, how many are there in a year? Round any intermediate calculations, if needed, to no less than six decimal places, and round your final answer to the nearest whole number.

Answers

The population of squirrels in the Louisiana forest growing monthly at a rate of 5% currently from 100, will be 182 after a year.

The final value of any quantity growing constantly at a particular rate is given as [tex]V = V_{0}e^{rt}[/tex] ,

where V is the final value, V₀ is the initial value, r is the rate of growth per time period, and t is the number of time periods.

The current population of squirrels (V₀) = 100.

The growth rate (r) = 5% per month.

The time period (t) = 1 year = 12 months.

Hence, the final population of squirrels (V), is given as:

[tex]V = V_{0}e^{rt}[/tex] ,

or, [tex]V = 100e^{(0.05*12)}[/tex] ,

or, [tex]V = 100e^{0.60}[/tex] ,

or, V = 100*1.822119,

or, V = 182.2119 ≈ 182.

Therefore, the population of squirrels in the Louisiana forest growing monthly at a rate of 5% currently from 100, will be 182 after a year.

Learn more about growth at

https://brainly.com/question/15241333

#SPJ4

Can 10 people be divided into two groups with a ratio of 1 : 2? Explain.

Answers

Answer:

no

Step-by-step explanation:

Can 10 people be divided into two groups with a ratio of 1 : 2?

Explain.

10 : 3 = 3.33

or 10 : 3 = 9 remaining 1

since it is not possible to divide a person into three parts, your answer is no

Change 9% to fraction? ​

Answers

9/100

Step-by-step explanation:

[tex]9\% = \frac{9}{100} [/tex]

♨ANSWER♥

9% as a Fraction is 9/100

...hope this helps...

_♡_mashi_♡_

Sharon drops a rubber ball from 160 feet every bounce sends the ball half as high as it was before. What is the total distance the ball has traveled from the moment it is dropped until it hits the floor for the fourth time

Answers

The total distance the ball has traveled from the moment it is dropped until it hits the floor for the fourth time is: 300 feet.

Total distance

First  bounce

160 feet

Second bounce

= 1/2×160 = 80 feet

Third bounce

= 1/2×80 = 40 feet

Fourth bounce

= 1/2×40 = 20 feet

Hence:

Total distance= (160 feet+ 80 feet + 40 feet+20 feet

Total distance= 300 feet

Therefore the total distance traveled by the ball is 300 feet.

Learn more about total distance here:https://brainly.com/question/11132018

#SPJ1

Use the given graph to determine the limit, if it exists. (4 points)
A coordinate graph is shown with an upward sloped line crossing the y axis at the origin that ends at the open point 3, 1, a closed point at 3, 7, and a horizontal line starting at the open point 3, 3.
Find limit as x approaches three from the right of f of x. .

Answers

Answer:  3

Explanation:

Refer to the graph below. It should be similar to what your teacher gave you, based off the description.

Since we're approaching 3 from the right side, this means we'll be working with the horizontal line portion. We could start at something like x = 3.2 and move closer to 3 by getting to x = 3.1 then x = 3.01 then x = 3.001 and so on. We never actually get to 3 itself.

As x gets closer to 3 from this direction, the y values are approaching 3 since every point on this horizontal line has the same y coordinate. Technically the y value is already at 3, but it's the same idea.

In terms of notation, we can write [tex]\displaystyle \lim_{x\to3^{+}}f(x) = 3[/tex]

The portion [tex]x \to 3^{+}[/tex] means we're approaching 3 from the positive side, aka the right hand side on the number line.

Simplify
1) 9/7+1/6
2) 1 1/6- 2 1/5
3) 4/7x8/4
4) 4/7* 8/4
you answer fast and correct I will mark you as a brainliest

Answers

Answer:

1 19/42

- 1 1/30

3&4 are both 1 1/7

Step-by-step explanation:

1) 1 19/42
2) -11/30
3) 1 1/7
4) 1 1/7

Find two digit number that is one less than a square number,and when doubled is one less than a square number.​

Answers

Answer is 24
24 is one less than 25 (5x5) and when doubled (24x2=48) is one less than a square number (7x7=49).

A plane has a maximum fuel capacity of 5,300 gallons. The airline requires that the pilot fly no more than 400 minutes before refueling. A one-way flight to a certain destination requires 2,000 gallons of fuel and 190 minutes of flight time. How many round trips can the pilot make before having to refuel?

A) 1

B) 2

C) 3

D) 4

Answers

Answer:

it is B

Step-by-step explanation:

that is because j3jejejdudduiwjsj

The requried pilot can make at least one round trip before having to refuel. Hence, the correct answer is: A) 1

To determine how many round trips the pilot can make before having to refuel, we need to consider both the fuel capacity and the flight time limit.

Given:

Maximum fuel capacity = 5,300 gallons

Flight time limit before refueling = 400 minutes

Fuel required for one-way flight = 2,000 gallons

Flight time for one-way flight = 190 minutes

First, let's calculate the maximum number of one-way trips the pilot can make based on the fuel capacity:

Max one-way trips based on fuel capacity = Maximum fuel capacity / Fuel required for one-way trip

Max one-way trips = 5,300 gallons / 2,000 gallons = 2.65 trips

Since we cannot have a fraction of a trip, we need to consider the flight time limit.

The total flight time for a round trip is twice the one-way flight time: 2 * 190 minutes = 380 minutes.

The pilot can fly a round trip as long as the flight time for the round trip is within the flight time limit. In this case, the round-trip flight time of 380 minutes is less than the flight time limit of 400 minutes.

Therefore, the pilot can make at least one round trip before having to refuel. Hence, the correct answer is: A) 1

Learn more about flight time here:

https://brainly.com/question/29482376

#SPJ4

Graph the function.
f(x)=5\cdot\left(\dfrac{4}{5}\right)^xf(x)=5⋅(
5
4

)
x
f, left parenthesis, x, right parenthesis, equals, 5, dot, left parenthesis, start fraction, 4, divided by, 5, end fraction, right parenthesis, start superscript, x, end superscript

Answers

Answer:

Please review my interpretation of the written expression.

Step-by-step explanation:

f(x) = 5*(4/5)^x    :  Is this correct?

If so, see attached graph.

One can calculate several points and then draw the graph, or use an online, and free, graphing app (DESMOS in this case).

Since the exponent is the variable, I'll pick a few vaules that are easy (for me) to calculate:

f(0) = 5*(4/5)^0

Anything raised to the 0th power is equal to 1, so we get 5*1 or 5 to make the point (1,5).

When x = 1, the fraction (4/5) remains (4/5) and when multiplied by 5, reduces to simply 4.  The second point is (1,4)

For the last point, I chose x to be 10.  5(4/5)^10 becomes 0.5.  The third point is (10,0.5)

We can see the line approaches 0 as x increases.

pls helpppp!!!!!!
What kind of compound inequality is….>>>>>look at pic

Answers

Answer:

  (d)  intersection

Step-by-step explanation:

When given two inequalities, you must determine whether the solution set must satisfy both, or must satisfy either. The key is found in the conjunction associated with the inequalities.

Conjunction

Here, the conjunction is "and," which means the solution must satisfy both inequalities. It also means, the solution set will be the intersection of the solution sets of the different inequalities.

The attached number line shows the first solution set in red, the second solution set in blue, and the "and" (intersection) of those in purple.

The conjunction "or" will signify the solution set is the union of those of the separate inequalities.

use the distrubutive property to simply -8(6x+3)

Answers

I think it is -48x-24
Other Questions
Find the largest rational number r for which 5r 33. XFrom the list, choose the word that best completes each of the sentences below.11. His outstanding performance in the classroom seemed toA a. locutionB) b. dictionC c. dictionaryd. elocution(E) e. indict(F) f. sublingualG g. dictateH) h. dictatorial11 OF 11 QUESTIONS REMAINIhis rather ordinary test scores. The electron configurations of some elements are given. based on the electron configurations, which elements can likely form ions with multiple charges? The combination of product lines offered by a manufacturer is called the firm's: Group of answer choices product matrix. product mix. total product offer. product portfolio. What is the ratio of 12 blue marbles and 20 red marbles Establishing a literary tradition in this country was connected to a sense of Nationalism during the period 1820-1865. True False Can someone help me with these geometry questions? I cant seem to understand them. If u find the answers, can u explain it too? Its urgent What are the medians for the height and weight distribution, respectively? what led to African nationalism Issue during World War II_____offered nations in region such as Africa in Latin America hope to gain independence from call Lynell powers that agreement stated that nations around the World head right to_____? What does letter C on the map represent?A) the location of where the first atomic bomb was droppedB) the last major Pacific battle of World War II, which the United States lost.C) the first major Pacific battle of World War II, which the United States wonD) the location of the most deadly battle of the Pacific War A Principal Investigator has submitted a new proposal to the NIH. Six months ago he accepted an invitation by the University of Texas Health Sciences Center at San Antonio (UTHSCSA) to do a presentation of his recent findings on breast cancer. UTHSCSA paid the Investigator $1000 to do the presentation and paid his travel expenses of $700. Under federal and UNM HSC COI Policy, the Investigator will need to do a financial conflict of interest disclosure of this information when he submits for this proposal to the NIH. True False Who wrote the first major theoretical paper (on what was known as the memory drum theory) from the discipline of kinesiology Which of the following expressions is equivalent to -g/h?Choose all the answers that apply:A. - -g/-hB. - g/-hC. None of the aboveplease help!! HELP HELP HELP HELPHELP HELP HELP HELP HELP HELP HELP HELP HELP HELP HELP HELP You are the coordinator for a program that is going to take place at night in a rectangular amphitheater in the mountains. You will have no access to any electricity, but you must be able to illuminate the entire grounds. You know the intensity of the light from a lantern varies inversely as the square of the distance from the lantern. Suppose the intensity is 90 when the distance is 5 m.a. Write an equation to model the situation.b. Solve for the constant of variation.c. Write the equation to model the situation using the constant () of variation.d. You have been given lanterns with 40 light intensity. Use your equation to solve for the distance from the lantern.e. You need to illuminate 225 km. How many meters do you need to light?f. How many lanterns will you need? Edit the following statement so it uses the constant named YEAR:newAge = currentAge + (2050 currentYear); Find the first five terms of the sequence described:a sub 1 = 3a sub n+1 = a sub n + 5 What are the x-intercepts for the graph below?A. (6,0), (-3,0)B. (-6,0), (-3,0)C. (-6,0), (3, 0)OD. (6,0), (3, 0) At noon, Bradley began steadily increasing thespeed of his car by 2 miles per hour every minute. At 12:15 p.m., he realized he was going15 miles per hour over the speed limit. If his speed at noon was 40 miles per hour, what was the speed limit at 12:15 p.m.?Please show work step by step